Italy - Math Olympiad Problem | A great approach

Sdílet
Vložit
  • čas přidán 9. 06. 2024
  • You should know this approach. Many goes WRONG!
    What do you think about this problem? Don't forget to like and subscribe to my channel for more helpful math tricks. Thanks For Watching!
    Check out my latest video (Can You Pass Harvard's Entrance Exam?): • Can You Pass Harvard's...
    If you're reading this ❤️. Thank You!
    Hello My Friend ! Welcome to my channel. I really appreciate it!
    ‪@higher_mathematics‬
    #maths #math

Komentáře • 26

  • @quantumbuddha777
    @quantumbuddha777 Před 15 dny +7

    More efficient method is to cube both sides and swap exponents on the left to get
    27^x=x^27, x=27

  • @user-yc4wh8lh8b
    @user-yc4wh8lh8b Před 14 dny

    Thanks, very good

  • @Math9_beheshti
    @Math9_beheshti Před 18 dny

    So useful

  • @DebdasBandyopadhyay-yq5jg

    Thanks 🙏 sir

  • @techintoshboy3144
    @techintoshboy3144 Před 15 dny +2

    Lowest approx value for x is 1.1508 by using graphical method

  • @Limited_Light
    @Limited_Light Před 20 dny +2

    Once you have 3^(x / 9) = x, considering other solutions besides natural numbers are not covered in this approach, why not the following:
    3^(x / 9) = 9 * (x / 9)
    u = x / 9
    3^u = 9u = 3^2 * u
    3^(u - 2) = u
    Trying u = 1, 2, and then 3, the first solution as a natural number gives us u = 3.
    x = 9 * u = 9 * 3 = 27.

    • @krishnakalidindi6671
      @krishnakalidindi6671 Před 16 dny

      How? 3^(x/9)=9*(x/9)

    • @Limited_Light
      @Limited_Light Před 16 dny

      @@krishnakalidindi6671 Did you read my entire comment?

    • @krishnakalidindi6671
      @krishnakalidindi6671 Před 16 dny

      @@Limited_Light yes.I could not understand your first step.

    • @Limited_Light
      @Limited_Light Před 16 dny

      @@krishnakalidindi6671 At time 1:44 , Higher Mathematics has the first step, that I began from, 3^(x / 9) = x.
      My change in direction uses x = 1 * x = (9 / 9) * x = 9 * (x / 9). (It's multiplication on the right side, not exponentiation.)
      Now that the exponent on the left has a similar form to something on the right, I substituted u for x / 9.

    • @krishnakalidindi6671
      @krishnakalidindi6671 Před 16 dny

      It's ok.very good.But it's a trial and error method.Good.

  • @prime423
    @prime423 Před 12 dny +4

    Outline a plan BEFORE you show a solution. The plan and your thinking are much more important than the grind. In addition, when you use the number one, a vertical line is preferred. In fact, I cant remember any teacher ever using your "1"for 1.

  • @Math9_beheshti
    @Math9_beheshti Před 18 dny

    Is there any number else that would have a rule like this

  • @nickcellino1503
    @nickcellino1503 Před 18 dny +4

    This difficult problem can be solved solely by the application of several laws of exponents. My AI software, MathTeacherXL, could not solve the problem even after I provided the answer! Instead of applying the laws of exponents, it tried using logarithms and approximation techniques, all to no avail.

  • @user-bi6nd4ty2q
    @user-bi6nd4ty2q Před 9 dny +1

    X=27

  • @RealQuInnMallory
    @RealQuInnMallory Před 20 dny

    (x ➖ 3x+3)

  • @baselinesweb
    @baselinesweb Před 13 dny

    Using Lambert W method produces 1.1508. Do you know how to use W method to get 27? Is it another branch?

    • @lagomoof
      @lagomoof Před 4 dny

      Yes. 27 comes from the -1 branch. There are infinitely many branches but only the -1 and 0 branches produce real results.

  • @AnkushRaj-bl5ye
    @AnkushRaj-bl5ye Před 6 dny

    I used the Lambert W function to solve this question, but I am getting only 1.151 and -0.8963(which is not possible) as solution and not 27, can anybody tell, why?
    Thank you.

  • @AbbasGuclu-jn3po
    @AbbasGuclu-jn3po Před 21 dnem

    x=27

  • @Maths__phyics
    @Maths__phyics Před 21 dnem

    This is the second time you share the same video

    • @DanDart
      @DanDart Před 20 dny

      What's up with that?

  • @DanDart
    @DanDart Před 20 dny

    This is the same as your 6 days ago video

  • @zeroone7500
    @zeroone7500 Před 18 dny

    your solving method is not complete, if you use Lambert W function there is another solution for x